If two permutations commute they are disjoint

Click For Summary
If two permutations α and β in S_n commute, it can be shown that β permutes the integers left fixed by α. The discussion highlights that while disjoint cycles commute, the reverse is not necessarily true, as demonstrated by a counterexample where two commuting permutations are not disjoint. The participants express confusion over the implications of the second part of the question, particularly regarding the condition that α must not equal any power of β. Ultimately, the conversation revolves around the complexities of permutation behavior in relation to commutativity and disjointness. The exploration of these properties reveals deeper insights into the structure of permutations in group theory.
3029298
Messages
56
Reaction score
0

Homework Statement


If \alpha,\beta\in S_n and if \alpha \beta = \beta \alpha, prove that \beta permutes those integers which are left fixed by \alpha. Show that \beta must be a power of \alpha when \alpha is a n-cycle.

The other way round is easy to see, since if two cycles are disjoint they do not do anything with the numbers permuted by the other cycle, hence they commute. But I don't know how to start when I want to prove the statement above... can anyone hint me?
 
Physics news on Phys.org
Hi 3029298! :smile:

(have an alpha: α and a beta: β :wink:)
3029298 said:
If \alpha,\beta\in S_n and if \alpha \beta = \beta \alpha, prove that \beta permutes those integers which are left fixed by \alpha.

But that's obviously not true …

put α = β: then α does not permute those integers which are left fixed by α. :confused:
 
Yes, I saw that as well, but I think (for the question to make sense) alpha must permute different elements than beta. But this makes the second part of the question questionable...
 
Hi. I'm new in the forum. I'd like to prove (or find counter example) the next statement.

If two permutations \alpha,\beta\in S_n such that \alpha\neq\beta^i for every i\in\mathbb{Z} commute then they're disjoint.

Do you think it's true?? I think it is.
 
Here is an example in S_5. Take \alpha=(12345), and \beta=(13524).

They commute:
(12345)(13524)=(14253),
(13524)(12345)=(14253).

But they are not disjoint (and neither is one a multiple of the other, re Unviaje).
 
oops I just realized that (12345)^2=(13524). I think you are right, Unviaje. I haven't found a proof yet though.
 
Question: A clock's minute hand has length 4 and its hour hand has length 3. What is the distance between the tips at the moment when it is increasing most rapidly?(Putnam Exam Question) Answer: Making assumption that both the hands moves at constant angular velocities, the answer is ## \sqrt{7} .## But don't you think this assumption is somewhat doubtful and wrong?

Similar threads

  • · Replies 4 ·
Replies
4
Views
2K
  • · Replies 3 ·
Replies
3
Views
2K
Replies
2
Views
2K
Replies
2
Views
2K
  • · Replies 4 ·
Replies
4
Views
3K
  • · Replies 1 ·
Replies
1
Views
2K
  • · Replies 3 ·
Replies
3
Views
2K
  • · Replies 11 ·
Replies
11
Views
2K
  • · Replies 2 ·
Replies
2
Views
2K
  • · Replies 2 ·
Replies
2
Views
1K